1984 AJHSME Problem 9

Revision as of 20:43, 31 January 2021 by Coolmath34 (talk | contribs) (Problem)

Solution

This product simplifies to: \[\frac{1}{2} \cdot \frac{2}{3} \dots \frac{9}{10}.\] Numerators and denominators cancel to yield the answer: $\boxed{\text{(A)} \frac{1}{10}}.$